Staffetta tdn

Numeri interi, razionali, divisibilità, equazioni diofantee, ...
Giulius
Messaggi: 58
Iscritto il: 02 apr 2009, 21:49
Località: Milano

Messaggio da Giulius »

Sinceramente non ho idea, in questo caso l'ho fatto solo per poter generalizzare a numeri di x cifre con x generico visto che
$ n+s(n)=2n_0+11n_1+101n_2...=\sum n_i(10^i+1) $, così ho potuto generalizzare il numero di cifre come estremo superiore della somma all'ultimo membro.
Sono stato cmq parecchio contorto, quella vagonata di conti alla fine è pressochè inutile credo...il succo della mia dimostrazione è che se definisco la successione $ {f_n} $ con $ f_i=i+s(i) $ allora $ f_{n+1}-f_n \le 2 $
Non escludo (anzi ne sono certo) che esista una soluzione molto più semplice e soprattutto più bella, forse proprio con quella roba sulle congruenze che dicevi tu.
Aboliamo il latino nei licei scientifici!
Giulius
Messaggi: 58
Iscritto il: 02 apr 2009, 21:49
Località: Milano

Problema 27

Messaggio da Giulius »

PROBLEMA 27:

Trovare tutti gli $ (x,y) \in \mathbb{N}^2 $ tali che:
$ xy+\frac{x^3+y^3}{3}=2007 $
Aboliamo il latino nei licei scientifici!
Alex90
Messaggi: 260
Iscritto il: 25 mag 2007, 13:49
Località: Perugia

Messaggio da Alex90 »

è possibile che non ci siano soluzioni?
Avatar utente
exodd
Messaggi: 728
Iscritto il: 09 mar 2007, 19:46
Località: sulle pendici della provincia più alta d'europa

Messaggio da exodd »

(3,18)
(18,3)
Tutto è possibile: L'impossibile richiede solo più tempo
julio14 ha scritto: jordan è in realtà l'origine e il fine di tutti i mali in $ \mathbb{N} $
EvaristeG ha scritto:Quindi la logica non ci capisce un'allegra e convergente mazza.
ispiratore del BTA

in geometry, angles are angels

"la traslazione non è altro che un'omotetia di centro infinito e k... molto strano"
Avatar utente
exodd
Messaggi: 728
Iscritto il: 09 mar 2007, 19:46
Località: sulle pendici della provincia più alta d'europa

Messaggio da exodd »

$ 3|x^3+y^3 $
$ 3|x+y $
ora tutto modulo 3
$ x=-y $
$ x^2=(x^3+y^3)/3 $
poniamo $ \displaystile{x} $ non multiplo di 3: allora 9 non divide $ \displaystile{x^3+y^3} $
impossibile, perchè
$ 3|x^3+y^3 $
implica
$ 9|x^3+y^3 $
quindi x è un multiplo di 3, così come y
$ x=3x' $
$ y=3y' $
$ x'y'+x'^3+y'^3=223 $
ragionando sempre modulo 3, uno tra x' e y' deve essere multiplo di 3.
ponendo x'=0,3,6, si scopre che le uniche soluzioni sono
(3,18)
(18,3)
Ultima modifica di exodd il 11 lug 2009, 13:30, modificato 1 volta in totale.
Tutto è possibile: L'impossibile richiede solo più tempo
julio14 ha scritto: jordan è in realtà l'origine e il fine di tutti i mali in $ \mathbb{N} $
EvaristeG ha scritto:Quindi la logica non ci capisce un'allegra e convergente mazza.
ispiratore del BTA

in geometry, angles are angels

"la traslazione non è altro che un'omotetia di centro infinito e k... molto strano"
ndp15
Messaggi: 598
Iscritto il: 18 gen 2007, 19:01

Messaggio da ndp15 »

exodd ha scritto: $ x'y'+x'^3+y'^3=233 $
È 223 non 233.
Comunque dovrebbe essere corretto, praticamente avevo fatto anche io il tuo stesso ragionamento (a parte il fatto $ x $ non multiplo di 3 su cui stavo ancora ragionando).
Avatar utente
exodd
Messaggi: 728
Iscritto il: 09 mar 2007, 19:46
Località: sulle pendici della provincia più alta d'europa

Messaggio da exodd »

corretto :roll:
Tutto è possibile: L'impossibile richiede solo più tempo
julio14 ha scritto: jordan è in realtà l'origine e il fine di tutti i mali in $ \mathbb{N} $
EvaristeG ha scritto:Quindi la logica non ci capisce un'allegra e convergente mazza.
ispiratore del BTA

in geometry, angles are angels

"la traslazione non è altro che un'omotetia di centro infinito e k... molto strano"
Avatar utente
exodd
Messaggi: 728
Iscritto il: 09 mar 2007, 19:46
Località: sulle pendici della provincia più alta d'europa

Messaggio da exodd »

PROBLEMA 28:

siano m e n numeri naturali tali che
$ mn|m^2+n^2+m $
provare che m è un quadrato perfetto

bonus question (own))
sia
$ d=gcd(m,n) $ $ b=n/d $
provare che
$ b|d^2+1 $
Tutto è possibile: L'impossibile richiede solo più tempo
julio14 ha scritto: jordan è in realtà l'origine e il fine di tutti i mali in $ \mathbb{N} $
EvaristeG ha scritto:Quindi la logica non ci capisce un'allegra e convergente mazza.
ispiratore del BTA

in geometry, angles are angels

"la traslazione non è altro che un'omotetia di centro infinito e k... molto strano"
Avatar utente
jordan
Messaggi: 3988
Iscritto il: 02 feb 2007, 21:19
Località: Pescara
Contatta:

Messaggio da jordan »

Addirittura come firma :lol:

Sia definita l'unica terna $ (a,b,d) \in \mathbb{N}_0^3 $ tale che $ (m,n)=d,(a,b)=1,ad=m,bd=n $, allora per ipotesi $ d^2ab \mid (da)^2+(db)^2+da\implies d \mid a $. Sia $ c \in \mathbb{N}_0 $ tale che $ cd=a $ allora $ bcd \mid (cd)^2+b^2+c $, con $ (b,cd)=1 $, in particolare $ c \mid b^2 $ e $ b \mid (cd)^2+c $. Per forza quindi $ c=1 $ (che risponde ad entrambe le domande).
The only goal of science is the honor of the human spirit.
Avatar utente
julio14
Messaggi: 1208
Iscritto il: 11 dic 2006, 18:52
Località: Berlino

Messaggio da julio14 »

[OT]
in firma exodd ha scritto:
julio14 ha scritto:jordan è in realtà l'origine e il fine di tutti i mali in $ ${\mathbb{N}} $
sniff sniff... :cry: :D
sono commosso... ora mi sento un po' TG anch'io
[/OT]
Avatar utente
jordan
Messaggi: 3988
Iscritto il: 02 feb 2007, 21:19
Località: Pescara
Contatta:

Messaggio da jordan »

Problema 29. (Austria 1989) Trovare tutti gli $ (a,b,c) \in \mathbb{Z}^3 $ tali che $ abc=1989 $ e $ a+b=89+c $.

@julio14: be, ognuno ha le sue aspirazioni :lol:
The only goal of science is the honor of the human spirit.
Avatar utente
iademarco
Messaggi: 264
Iscritto il: 10 dic 2008, 22:12
Località: Campobasso

Messaggio da iademarco »

jordan ha scritto:Problema 29. (Austria 1989) Trovare tutti gli $ (a,b,c) \in \mathbb{Z}^3 $ tali che $ abc=1989 $ e $ a+b=89+c $.
Questo è il mio primo messaggio nella staffetta...vediamo un po' che ci esce :roll:
Abbiamo $ 1989=3^{2}\cdot{13}\cdot{17} $;

ora le possibili terne sono:
1) $ 1-3-663\equiv1-3-6\mod 9 $
2) $ 1-9-221\equiv 1-0-5\mod 9 $
3) $ 1-39-51\equiv1-3-6\mod 9 $
4) $ 3-3-221\equiv3-3-5\mod 9 $
5) $ 3-39-17\equiv3-3-8\mod 9 $
6) $ 3-51-13\equiv3-6-4\mod 9 $
7) $ 9-13-17\equiv0-4-8\mod 9 $
8 ) $ 1-13-153\equiv1-4-0\mod 9 $
9) $ 1-17-117\equiv1-8-0\mod 9 $
(non scrivo anche quelle con numeri negativi perchè non c'è n'è bisogno)

Dato che $ 89\equiv8\mod 9 $, $ a+b-c\equiv8\mod 9 $, con delle semplici e corte verifiche vediamo che i soli che possono essere $ \equiv8\mod 9 $ sono la
1) con 6+3-1
2) impossibile
3) con 6+3-1
4) con 5-3-3
5) con 8-3+3
6) con 6-4-3
7) impossibile
8 ) impossibile
9) impossibile

Ora per verifica diretta (è breve quindi possiamo) vediamo che la sola terna possibile è la 3° con 6+3-1 ovvero 51+39-1=89, cioè (51,39,1).
Ultima modifica di iademarco il 13 lug 2009, 00:09, modificato 2 volte in totale.
"Il lemma fondamentale: se vi danno un esercizio è perchè potete farlo; se potete farlo è perchè è proprio facile; se è proprio facile è perchè servono delle cose che sapete; le cose che sapete sono pochissime, quindi avete da cercare in un insieme piccolissimo di cose" Michele Barsanti


[quote="julio14"]
jordan è in realtà l'origine e il fine di tutti i mali in [tex]\mathbb{N}[/tex][/quote]
Avatar utente
exodd
Messaggi: 728
Iscritto il: 09 mar 2007, 19:46
Località: sulle pendici della provincia più alta d'europa

Messaggio da exodd »

iademarco ha scritto:bili terne sono:

5) $ 3-39-17\equiv3-5-8\mod 9 $

5) impossibile
è 3 3 8
con 8-3+3=8
Tutto è possibile: L'impossibile richiede solo più tempo
julio14 ha scritto: jordan è in realtà l'origine e il fine di tutti i mali in $ \mathbb{N} $
EvaristeG ha scritto:Quindi la logica non ci capisce un'allegra e convergente mazza.
ispiratore del BTA

in geometry, angles are angels

"la traslazione non è altro che un'omotetia di centro infinito e k... molto strano"
Avatar utente
iademarco
Messaggi: 264
Iscritto il: 10 dic 2008, 22:12
Località: Campobasso

Messaggio da iademarco »

Siano a,b due interi positivi tali che $ \frac{a^{2}+b^{2}+1}{ab} $ è intero.
Mostrare che $ \frac{a^{2}+b^{2}+1}{ab}=3 $

(comunque mi è stato suggerito da un necrofilo che conoscete mooolto bene) :lol:
"Il lemma fondamentale: se vi danno un esercizio è perchè potete farlo; se potete farlo è perchè è proprio facile; se è proprio facile è perchè servono delle cose che sapete; le cose che sapete sono pochissime, quindi avete da cercare in un insieme piccolissimo di cose" Michele Barsanti


[quote="julio14"]
jordan è in realtà l'origine e il fine di tutti i mali in [tex]\mathbb{N}[/tex][/quote]
travelsga
Messaggi: 39
Iscritto il: 30 giu 2008, 13:40
Località: Carrara

Messaggio da travelsga »

$ \displaystyle\frac{a^2+b^2+1}{ab}=k $ (1).
Dimostro che necessariamente si ha $ a=b $. Sia $ (A,B) $ una soluzione della (1) che minimizza $ A+B $ e sia wlog $ A>B $ considero quindi la quadratica $ a^2-kBa+B^2+1=0 $; le soluzioni di tale equazione sono $ a_1=A $ e, dalle formule di Vieta, $ \displaystyle a_2=kB-A=\frac{B^2+1}{A} $, ma $ \displaystyle A+\frac{B^2+1}{A}<A+B\Longleftrightarrow B^2+1<AB $, quest'ultima disuguaglianza è verificata in quanto si è assunto $ A>B $. Abbiamo pertanto trovato una coppia $ (A,\frac{B^2+1}{A}) $ con somma minore, assurdo. Deve essere $ a=b $, ma allora $ \displaystyle\frac{a^2+b^2+1}{ab}=\frac{2a^2+1}{a^2}=2+\frac{1}{a^2} $ , questa quantità risulta intera se e solo se $ a^2=1 $.
Rispondi